Đến nội dung

Hình ảnh

Chứng minh rằng: $M\geq \frac{4}{n}$.

cuctritohop

  • Please log in to reply
Chủ đề này có 1 trả lời

#1
Hoang72

Hoang72

    Thiếu úy

  • Điều hành viên OLYMPIC
  • 539 Bài viết

Mời mọi người cùng góp lời giải:

Cho $a_1,a_2,...,a_n$ là các số thực có tổng bằng $0$ $(n\in\mathbb N; n\geq 2)$, trong đó tồn tại một số bằng $1$.

Đặt $M = \max\left\{|a_1 - a_2|, |a_2-a_3|,...,|a_n-a_1|\right\}$.

Chứng minh rằng: $M\geq \frac{4}{n}$.



#2
nhungvienkimcuong

nhungvienkimcuong

    Thiếu úy

  • Hiệp sỹ
  • 669 Bài viết

Mời mọi người cùng góp lời giải:

Cho $a_1,a_2,...,a_n$ là các số thực có tổng bằng $0$ $(n\in\mathbb N; n\geq 2)$, trong đó tồn tại một số bằng $1$.

Đặt $M = \max\left\{|a_1 - a_2|, |a_2-a_3|,...,|a_n-a_1|\right\}$.

Chứng minh rằng: $M\geq \frac{4}{n}$.

Giả sử $a_1=1$ và $M<\frac{4}{n}$, nghĩa là $|a_k-a_{k+1}|<\frac{4}{n}$ với mọi $k=\overline{1,n}$ (xem như $a_{n+1}=a_1$).

$\bullet$ Chứng minh $a_k>1-\frac{4}{n}\min(k-1,n+1-k)$ với mọi $k=\overline{2,n}$.

Cộng $n-1$ bất đẳng thức trên ta có

$$\sum_{k=2}^{n}a_k>n-1-\frac{4}{n}\sum_{k=2}^{n}\min(k-1,n+1-k).$$

Vì $\sum_{k=2}^{n}a_k=-a_1=-1$ nên bất đẳng cuối tương đương với $\sum_{k=2}^{n}\min(k-1,n+1-k)>\frac{n^2}{4}$. Ta sẽ chỉ ra mâu thuẫn bằng cách chứng tỏ nhận xét sau

$\bullet$ Chứng minh $\sum_{k=2}^{n}\min(k-1,n+1-k)\le \frac{n^2}{4}$.


Đừng khóc vì chuyện đã kết thúc hãy cười vì chuyện đã xảy ra ~O) 
Thật kì lạ anh không thể nhớ đến tên mình mà chỉ nhớ đến tên em :wub:
Chúa tạo ra vũ trụ của con người còn em tạo ra vũ trụ của anh :ukliam2:





1 người đang xem chủ đề

0 thành viên, 1 khách, 0 thành viên ẩn danh